I can't understand logical implication












19












$begingroup$


I just started studying logic (high school)
anyway...for the truth table of logical implication



If sentence $A$ is true and $B$ is true then $Aimplies B$ is true.



does that mean if $A$ and $B$ are both true
then there is a way to prove $B$ is true from $A$,
always?



the same for if $A$ is false can you get anything either True or false proved from this $A$?










share|cite|improve this question











$endgroup$








  • 10




    $begingroup$
    @gebruiker Yes, you can.
    $endgroup$
    – Git Gud
    Sep 27 '14 at 10:10






  • 3




    $begingroup$
    @gebruiker - Why would you think you can't ?
    $endgroup$
    – Belgi
    Sep 27 '14 at 10:20






  • 3




    $begingroup$
    @Belgi Because I own a dog is true, and I am a man is true. But owning a dog does not imply being a man. I know women who own dogs too. Or am I overlooking something here?
    $endgroup$
    – gebruiker
    Sep 27 '14 at 10:32






  • 10




    $begingroup$
    @gebruiker "(gebruiker owns a dog) implies (gebruiker is a man)" is true. "(X owns a dog) implies (X is a man)" is not true for all X
    $endgroup$
    – Ben Aaronson
    Sep 27 '14 at 20:33






  • 6




    $begingroup$
    $Aimplies B ;;;;$ means $;;; operatorname{truthvalue}(A) leq operatorname{truthvalue}(B) ;;;$. $;;;;;;;$
    $endgroup$
    – user57159
    Sep 28 '14 at 2:01
















19












$begingroup$


I just started studying logic (high school)
anyway...for the truth table of logical implication



If sentence $A$ is true and $B$ is true then $Aimplies B$ is true.



does that mean if $A$ and $B$ are both true
then there is a way to prove $B$ is true from $A$,
always?



the same for if $A$ is false can you get anything either True or false proved from this $A$?










share|cite|improve this question











$endgroup$








  • 10




    $begingroup$
    @gebruiker Yes, you can.
    $endgroup$
    – Git Gud
    Sep 27 '14 at 10:10






  • 3




    $begingroup$
    @gebruiker - Why would you think you can't ?
    $endgroup$
    – Belgi
    Sep 27 '14 at 10:20






  • 3




    $begingroup$
    @Belgi Because I own a dog is true, and I am a man is true. But owning a dog does not imply being a man. I know women who own dogs too. Or am I overlooking something here?
    $endgroup$
    – gebruiker
    Sep 27 '14 at 10:32






  • 10




    $begingroup$
    @gebruiker "(gebruiker owns a dog) implies (gebruiker is a man)" is true. "(X owns a dog) implies (X is a man)" is not true for all X
    $endgroup$
    – Ben Aaronson
    Sep 27 '14 at 20:33






  • 6




    $begingroup$
    $Aimplies B ;;;;$ means $;;; operatorname{truthvalue}(A) leq operatorname{truthvalue}(B) ;;;$. $;;;;;;;$
    $endgroup$
    – user57159
    Sep 28 '14 at 2:01














19












19








19


6



$begingroup$


I just started studying logic (high school)
anyway...for the truth table of logical implication



If sentence $A$ is true and $B$ is true then $Aimplies B$ is true.



does that mean if $A$ and $B$ are both true
then there is a way to prove $B$ is true from $A$,
always?



the same for if $A$ is false can you get anything either True or false proved from this $A$?










share|cite|improve this question











$endgroup$




I just started studying logic (high school)
anyway...for the truth table of logical implication



If sentence $A$ is true and $B$ is true then $Aimplies B$ is true.



does that mean if $A$ and $B$ are both true
then there is a way to prove $B$ is true from $A$,
always?



the same for if $A$ is false can you get anything either True or false proved from this $A$?







logic intuition propositional-calculus






share|cite|improve this question















share|cite|improve this question













share|cite|improve this question




share|cite|improve this question








edited Sep 27 '14 at 10:06









Hanul Jeon

17.5k42780




17.5k42780










asked Sep 27 '14 at 10:03









linalina

96114




96114








  • 10




    $begingroup$
    @gebruiker Yes, you can.
    $endgroup$
    – Git Gud
    Sep 27 '14 at 10:10






  • 3




    $begingroup$
    @gebruiker - Why would you think you can't ?
    $endgroup$
    – Belgi
    Sep 27 '14 at 10:20






  • 3




    $begingroup$
    @Belgi Because I own a dog is true, and I am a man is true. But owning a dog does not imply being a man. I know women who own dogs too. Or am I overlooking something here?
    $endgroup$
    – gebruiker
    Sep 27 '14 at 10:32






  • 10




    $begingroup$
    @gebruiker "(gebruiker owns a dog) implies (gebruiker is a man)" is true. "(X owns a dog) implies (X is a man)" is not true for all X
    $endgroup$
    – Ben Aaronson
    Sep 27 '14 at 20:33






  • 6




    $begingroup$
    $Aimplies B ;;;;$ means $;;; operatorname{truthvalue}(A) leq operatorname{truthvalue}(B) ;;;$. $;;;;;;;$
    $endgroup$
    – user57159
    Sep 28 '14 at 2:01














  • 10




    $begingroup$
    @gebruiker Yes, you can.
    $endgroup$
    – Git Gud
    Sep 27 '14 at 10:10






  • 3




    $begingroup$
    @gebruiker - Why would you think you can't ?
    $endgroup$
    – Belgi
    Sep 27 '14 at 10:20






  • 3




    $begingroup$
    @Belgi Because I own a dog is true, and I am a man is true. But owning a dog does not imply being a man. I know women who own dogs too. Or am I overlooking something here?
    $endgroup$
    – gebruiker
    Sep 27 '14 at 10:32






  • 10




    $begingroup$
    @gebruiker "(gebruiker owns a dog) implies (gebruiker is a man)" is true. "(X owns a dog) implies (X is a man)" is not true for all X
    $endgroup$
    – Ben Aaronson
    Sep 27 '14 at 20:33






  • 6




    $begingroup$
    $Aimplies B ;;;;$ means $;;; operatorname{truthvalue}(A) leq operatorname{truthvalue}(B) ;;;$. $;;;;;;;$
    $endgroup$
    – user57159
    Sep 28 '14 at 2:01








10




10




$begingroup$
@gebruiker Yes, you can.
$endgroup$
– Git Gud
Sep 27 '14 at 10:10




$begingroup$
@gebruiker Yes, you can.
$endgroup$
– Git Gud
Sep 27 '14 at 10:10




3




3




$begingroup$
@gebruiker - Why would you think you can't ?
$endgroup$
– Belgi
Sep 27 '14 at 10:20




$begingroup$
@gebruiker - Why would you think you can't ?
$endgroup$
– Belgi
Sep 27 '14 at 10:20




3




3




$begingroup$
@Belgi Because I own a dog is true, and I am a man is true. But owning a dog does not imply being a man. I know women who own dogs too. Or am I overlooking something here?
$endgroup$
– gebruiker
Sep 27 '14 at 10:32




$begingroup$
@Belgi Because I own a dog is true, and I am a man is true. But owning a dog does not imply being a man. I know women who own dogs too. Or am I overlooking something here?
$endgroup$
– gebruiker
Sep 27 '14 at 10:32




10




10




$begingroup$
@gebruiker "(gebruiker owns a dog) implies (gebruiker is a man)" is true. "(X owns a dog) implies (X is a man)" is not true for all X
$endgroup$
– Ben Aaronson
Sep 27 '14 at 20:33




$begingroup$
@gebruiker "(gebruiker owns a dog) implies (gebruiker is a man)" is true. "(X owns a dog) implies (X is a man)" is not true for all X
$endgroup$
– Ben Aaronson
Sep 27 '14 at 20:33




6




6




$begingroup$
$Aimplies B ;;;;$ means $;;; operatorname{truthvalue}(A) leq operatorname{truthvalue}(B) ;;;$. $;;;;;;;$
$endgroup$
– user57159
Sep 28 '14 at 2:01




$begingroup$
$Aimplies B ;;;;$ means $;;; operatorname{truthvalue}(A) leq operatorname{truthvalue}(B) ;;;$. $;;;;;;;$
$endgroup$
– user57159
Sep 28 '14 at 2:01










11 Answers
11






active

oldest

votes


















33












$begingroup$

As a logical proposition, the material conditional $A implies B$ is a very weak one: as you've noticed, it's very easy to satisfy it just by accident. In fact, this happens whenever $A$ is false, or whenever $B$ is true. Thus, merely observing that $A implies B$, for some specific $A$ and $B$, says very little.



Instead, the usefulness of implication lies in the fact that, precisely because of its weakness, it is often possible to assert $A implies B$ as a universal statement (either an axiom or a provable theorem) that holds for any valuation of any free variables mentioned in the propositions $A$ and $B$.



For example, consider the statement: $$x > 2 ;land; x text{ is prime} implies x text{ is odd}.$$ Merely observing that this statement holds for some $x$ says very little — there are plenty of numbers for which it is trivially true, either because they are odd, or because they are not primes greater than 2. What makes this statement useful is that we can prove that it holds for all $x$ — there isn't a single number which would be greater than 2 and prime, but not odd.






share|cite|improve this answer











$endgroup$













  • $begingroup$
    This is the answer that gets to the heart of the matter. +1.
    $endgroup$
    – 6005
    Sep 27 '14 at 16:18



















11












$begingroup$

One way to understand implication is to remember that $ARightarrow B$ is equivalent to $neg A lor B$. If you understand negation ($neg$) and disjunction ($lor$), then you understand implication.






share|cite|improve this answer









$endgroup$









  • 7




    $begingroup$
    I thought this was a strange equivalence, until I noticed myself saying things like: "I didn't know that or I would have done this"
    $endgroup$
    – Justin
    Sep 28 '14 at 4:47








  • 1




    $begingroup$
    I find $neg[Aland neg B]$ easier to work with.
    $endgroup$
    – Dan Christensen
    Sep 29 '14 at 15:39



















5












$begingroup$

Look at $A$ and $B$ as something that is either false or true. For
example let $A$ be the event that tomorrow is Tuesday and let $B$
be the event that the day after tomorrow is Wednesday.



Look at
$$
Aimplies B
$$



as a promise - if $A$ is true then so is $B$.



In our example, if $A$ is true then indeed so is $B$ and so the
implication $Aimplies B$ is true.



However, now consider $C$ as the statement that tomorrow is Friday,
and I state
$$
Cimplies B
$$



that is - I promise you that if $C$ will happen so will $B$.



Tomorrow is not Friday (at the time of writing), and so $C$ is false,
regardless of if $B$ is false or true - my promise was kept.



Now regarding the terminology of proofing $B$ is both $A,B$ are
true. Note that statements like
$$
text{My cat walks on four}implies1+1=2
$$



is true, since both are true, but what would it mean to prove $B$ from $A$
?






share|cite|improve this answer











$endgroup$





















    2












    $begingroup$

    I would say that $A$ being true and $B$ being true does not mean you can always prove (deduce) $B$ from $A$.



    Here's an example. A: Alice lives in Atlanta. B: Bob lives in Boston. Even if these are both true, there is no (apparent) relationship. So you can't logically deduce $B$ from $A$ even though $ARightarrow B$ is true in this case.



    I guess this means that if you can logically deduce statement $Q$ from statement $P$, then $PRightarrow Q$ is true; but knowing $PRightarrow Q$ is true does not guarantee the existence of a deduction of $Q$ from the assumption of $P$.



    Logical implication is a defined logical connective, so as long as $P$ and $Q$ have truth values (true or false), so does $PRightarrow Q$.






    share|cite|improve this answer









    $endgroup$













    • $begingroup$
      You can logically deduce B from A in that example. Let's assume A is true. B is true. QED. Yes, this counts, unless you really meant A implies B and also not not-A implies not-B. But if you meant that, you should have said so.
      $endgroup$
      – djechlin
      Sep 28 '14 at 2:00










    • $begingroup$
      Your 3rd paragraph is incorrect. Suppose $PRightarrow Q$ is given, we construct a proof of $Q$ to $P$: Assume $P$, assume $neg Q$, then by the truth table for $PRightarrow Q$ (or by modes ponens) $Q$. Hence $Q$ and $neg Q$, a contradiction. Hence our assumption $neg Q$ was false.
      $endgroup$
      – James
      Sep 29 '14 at 16:41










    • $begingroup$
      My point is that sometimes two statements, $P$ and $Q$ are unrelated. Now $PRightarrow Q$ can perfectly well be true (provided $P$ is false or $Q$ is true), and yet knowing that $P$ is true does not help us determine (deduce) that $Q$ is true, because the statements are unrelated.
      $endgroup$
      – paw88789
      Sep 29 '14 at 17:43










    • $begingroup$
      @paw88789 But when you know $P$, then to quote you "knowing $PRightarrow Q$ is true does not guarantee the existence of a deduction of $Q$ from the assumption that $P$." This is wrong, I just gave a deduction.
      $endgroup$
      – James
      Sep 29 '14 at 18:42



















    2












    $begingroup$

    Maybe it's more clear if we separate the logical operator meaning of implication from its logical statement meaning.



    When we use it as a logical operator, we conceive it simply as an entity, that given two logical values (thus true or false), produces a third logical value, using a common defined rule (its truth table).

    So it makes perfect sense to say $A implies B = true$ if $A = B = true$, and we don't concern about what actually proposition $A$ and $B$ means, we care exclusively about their logical values.



    It's different when we use implication as a logical statement.

    In this case we really say something about the meaning of the propositions involved in our statement. So, while proposition $A = My;cat;is;black$ is true, and proposition $B = I;am;hungry$ is true as well, $A implies B$ is not a valid statement.

    Such meaning is linked to set theory and formal logic. Using implication in this context means that you can infer $B$ from $A$, in a way called modus ponens.






    share|cite|improve this answer











    $endgroup$





















      2












      $begingroup$

      Implication can be understood as thinking in NECESSARY and SUFFICIENT conditions. A good example is to think about someone who has born in Dallas, Texas.



      Proposition P could stand for "Someone who was born in Dallas"



      Proposition Q could stand for "Someone who is texan"



      Truth table for implication is:



      enter image description here



      So, in this example is easy to check line by line the validity of truth table above:




      • Who was born in Dallas is therefore texan; So the first implication is TRUE.


      • Who was born in Dallas MUST be texan. So, saying that someone who was born in Dallas, in the state of Texas, and is not texan is FALSE;


      • Saying who wasn't born in Dallas but is texan anyway (take someone who has born in Houston, for example) is a TRUE statement;


      • At last, who wasn't born in Dallas AND also is NOT texan is also TRUE.



      So, thinking in terms of NECESSARY (to be texan is a necessary condition for who was born in Dallas) and SUFFICIENT (to born in Dallas is a sufficient condition to be texan) conditions ease the process to translate implication to natural language.



      Other examples of propositions containing necessary and sufficient conditions are:




      • P = To be a priest / Q = To believe in God;

      • P = To be a Police officer / Q = To carry a gun;

      • P = The number is divisible by 4 / Q = The number is even.






      share|cite|improve this answer











      $endgroup$





















        1












        $begingroup$

        The problem is understanding that there's a difference between the everyday mathematical meaning of "implies" - "I can prove it (or someone can)" - and the formal logical meaning expressed by the truth table. This may help: In classical logic, why is $(pRightarrow q)$ True if $p$ is False and $q$ is True?.






        share|cite|improve this answer











        $endgroup$





















          0












          $begingroup$


          If sentence $A$ is true and $B$ is true then $Aimplies B$ is true.




          Yes. In mathematics and logic as widely applied, $Aimplies B$ is equivalent to $neg[Alandneg B]$.



          So, if $A$ is true and $B$ is true, then $neg[Alandneg B]$ and hence $Aimplies B$ will also be true.




          Does that mean if $A$ and $B$ are both true then there is a way to prove $B$ is true from $A$, always?




          Yes. Here is an informal proof:




          1. Suppose $A land B$.


          2. Suppose $A$.


          3. $B$ (from 1)


          4. $Aimplies B$ (conclusion from 2 and 3)


          5. $Aland B implies [Aimplies B]$ (conclusion from 1 and 4)



          Alternatively, prove (5) using truth table.




          The same for if $A$ is false can you get anything either True or false proved from this $A$?




          Yes. Informal proof:




          1. Suppose $neg A$


          2. Suppose $Aland neg B$


          3. $Aland neg A$ (contradiction from 1 and 2)


          4. $neg[Aland neg B]$ (by contradiciton from 2 and 3)


          5. $Aimplies B$ from (equivalently from 4)


          6. $neg Aimplies [Aimplies B] $ (conclusion from 1 and 5)



          Alternatively, prove (6) using a truth table.





          That the above may seem counter-intuitive may stem from the fact that implication is often confused with temporal notions of causality. The statement, "If it is raining, then it is cloudy" on its own does not mean that rain always causes clouds, or that clouds always cause rain. If means only that, at a given instant in time, its is not the case that it is raining and not cloudy.






          share|cite|improve this answer











          $endgroup$





















            0












            $begingroup$

            The sentence $Aimplies B$ is not a sentence about proving anything. It is a compound sentence, but otherwise just like the sentences $A$ and $B$. For the truth table, the compound sentence got to have truth values like $A$ and $B$ has, and it has to depend on these sentences truth values. The values for this compound sentence is the same as for the compound statement $neg A vee B$.






            share|cite|improve this answer









            $endgroup$





















              0












              $begingroup$

              This is the way I understand it (I make no guarantee this is considered a correct interpretation).



              the truth value of an implies statement is basically 'innocent until proven guilty', or 'my statement is true so long as the proposition values don't disprove it'.



              So with an example, propositions P and Q, with the statement P implies Q. Lets say proposition P is 'is a man' and proposition Q is 'lives in japan', so our implication statement is basically saying 'all men live in japan'.



              The truth table:



              truth table



              is basically saying, for any example where P and Q are true, we haven't disproved our statement that P -> Q, and for any example where P is false we haven't disproved our statement, only if we have an example of P being true and Q being false is our statement shown to be false.



              So imagine there are only 4 humans in existence, we look at each of them and categorise them in our truth table. By doing this we can work out if our statement that P -> Q is true. We find 2 of the people are men, and that they do live in Japan, OK good, our statement still holds true, the 3rd person is a woman, we don't care if she lives in Japan or not because in either case our statement about men is still holding true.



              A woman living in or outside japan has no bearing on the truth of our statement about men living in japan.



              The 4th person then will decide if our statement is true or not, if the 4th person is a man living in japan, or a woman, then is it logically true to say that being a man implies that you live in japan.



              Why is this useful? well the truth table itself doesn't help us prove the truth of our statement by itself, what it does is clearly set out what conditions are relevant to the truth of our statement - i.e. that we don't care about women, and that only a man not living in japan will falsify our statement.



              Of course, you could make a statement like 'all green men live on mars'. This is a true statement because we cannot verify who lives on mars, and we don't have any examples of green men. But this is whats called a "vacuous truth", it has no bearing on reality for us because the propositions are intangible.






              share|cite|improve this answer









              $endgroup$





















                -2












                $begingroup$

                This illustration of logical implication might help:



                a) When you truly understand logical implication,

                b) then you’ll be a happy person.



                You may be a happy person for other reasons. But if you’re unhappy, then certainly you don’t truly understand logical implication. :-)



                Truth table:

                a b a⇒b

                F F T

                F T T

                T F F

                T T T



                I hope this somewhat intuitive example helps.






                share|cite|improve this answer











                $endgroup$













                  Your Answer





                  StackExchange.ifUsing("editor", function () {
                  return StackExchange.using("mathjaxEditing", function () {
                  StackExchange.MarkdownEditor.creationCallbacks.add(function (editor, postfix) {
                  StackExchange.mathjaxEditing.prepareWmdForMathJax(editor, postfix, [["$", "$"], ["\\(","\\)"]]);
                  });
                  });
                  }, "mathjax-editing");

                  StackExchange.ready(function() {
                  var channelOptions = {
                  tags: "".split(" "),
                  id: "69"
                  };
                  initTagRenderer("".split(" "), "".split(" "), channelOptions);

                  StackExchange.using("externalEditor", function() {
                  // Have to fire editor after snippets, if snippets enabled
                  if (StackExchange.settings.snippets.snippetsEnabled) {
                  StackExchange.using("snippets", function() {
                  createEditor();
                  });
                  }
                  else {
                  createEditor();
                  }
                  });

                  function createEditor() {
                  StackExchange.prepareEditor({
                  heartbeatType: 'answer',
                  autoActivateHeartbeat: false,
                  convertImagesToLinks: true,
                  noModals: true,
                  showLowRepImageUploadWarning: true,
                  reputationToPostImages: 10,
                  bindNavPrevention: true,
                  postfix: "",
                  imageUploader: {
                  brandingHtml: "Powered by u003ca class="icon-imgur-white" href="https://imgur.com/"u003eu003c/au003e",
                  contentPolicyHtml: "User contributions licensed under u003ca href="https://creativecommons.org/licenses/by-sa/3.0/"u003ecc by-sa 3.0 with attribution requiredu003c/au003e u003ca href="https://stackoverflow.com/legal/content-policy"u003e(content policy)u003c/au003e",
                  allowUrls: true
                  },
                  noCode: true, onDemand: true,
                  discardSelector: ".discard-answer"
                  ,immediatelyShowMarkdownHelp:true
                  });


                  }
                  });














                  draft saved

                  draft discarded


















                  StackExchange.ready(
                  function () {
                  StackExchange.openid.initPostLogin('.new-post-login', 'https%3a%2f%2fmath.stackexchange.com%2fquestions%2f947989%2fi-cant-understand-logical-implication%23new-answer', 'question_page');
                  }
                  );

                  Post as a guest















                  Required, but never shown

























                  11 Answers
                  11






                  active

                  oldest

                  votes








                  11 Answers
                  11






                  active

                  oldest

                  votes









                  active

                  oldest

                  votes






                  active

                  oldest

                  votes









                  33












                  $begingroup$

                  As a logical proposition, the material conditional $A implies B$ is a very weak one: as you've noticed, it's very easy to satisfy it just by accident. In fact, this happens whenever $A$ is false, or whenever $B$ is true. Thus, merely observing that $A implies B$, for some specific $A$ and $B$, says very little.



                  Instead, the usefulness of implication lies in the fact that, precisely because of its weakness, it is often possible to assert $A implies B$ as a universal statement (either an axiom or a provable theorem) that holds for any valuation of any free variables mentioned in the propositions $A$ and $B$.



                  For example, consider the statement: $$x > 2 ;land; x text{ is prime} implies x text{ is odd}.$$ Merely observing that this statement holds for some $x$ says very little — there are plenty of numbers for which it is trivially true, either because they are odd, or because they are not primes greater than 2. What makes this statement useful is that we can prove that it holds for all $x$ — there isn't a single number which would be greater than 2 and prime, but not odd.






                  share|cite|improve this answer











                  $endgroup$













                  • $begingroup$
                    This is the answer that gets to the heart of the matter. +1.
                    $endgroup$
                    – 6005
                    Sep 27 '14 at 16:18
















                  33












                  $begingroup$

                  As a logical proposition, the material conditional $A implies B$ is a very weak one: as you've noticed, it's very easy to satisfy it just by accident. In fact, this happens whenever $A$ is false, or whenever $B$ is true. Thus, merely observing that $A implies B$, for some specific $A$ and $B$, says very little.



                  Instead, the usefulness of implication lies in the fact that, precisely because of its weakness, it is often possible to assert $A implies B$ as a universal statement (either an axiom or a provable theorem) that holds for any valuation of any free variables mentioned in the propositions $A$ and $B$.



                  For example, consider the statement: $$x > 2 ;land; x text{ is prime} implies x text{ is odd}.$$ Merely observing that this statement holds for some $x$ says very little — there are plenty of numbers for which it is trivially true, either because they are odd, or because they are not primes greater than 2. What makes this statement useful is that we can prove that it holds for all $x$ — there isn't a single number which would be greater than 2 and prime, but not odd.






                  share|cite|improve this answer











                  $endgroup$













                  • $begingroup$
                    This is the answer that gets to the heart of the matter. +1.
                    $endgroup$
                    – 6005
                    Sep 27 '14 at 16:18














                  33












                  33








                  33





                  $begingroup$

                  As a logical proposition, the material conditional $A implies B$ is a very weak one: as you've noticed, it's very easy to satisfy it just by accident. In fact, this happens whenever $A$ is false, or whenever $B$ is true. Thus, merely observing that $A implies B$, for some specific $A$ and $B$, says very little.



                  Instead, the usefulness of implication lies in the fact that, precisely because of its weakness, it is often possible to assert $A implies B$ as a universal statement (either an axiom or a provable theorem) that holds for any valuation of any free variables mentioned in the propositions $A$ and $B$.



                  For example, consider the statement: $$x > 2 ;land; x text{ is prime} implies x text{ is odd}.$$ Merely observing that this statement holds for some $x$ says very little — there are plenty of numbers for which it is trivially true, either because they are odd, or because they are not primes greater than 2. What makes this statement useful is that we can prove that it holds for all $x$ — there isn't a single number which would be greater than 2 and prime, but not odd.






                  share|cite|improve this answer











                  $endgroup$



                  As a logical proposition, the material conditional $A implies B$ is a very weak one: as you've noticed, it's very easy to satisfy it just by accident. In fact, this happens whenever $A$ is false, or whenever $B$ is true. Thus, merely observing that $A implies B$, for some specific $A$ and $B$, says very little.



                  Instead, the usefulness of implication lies in the fact that, precisely because of its weakness, it is often possible to assert $A implies B$ as a universal statement (either an axiom or a provable theorem) that holds for any valuation of any free variables mentioned in the propositions $A$ and $B$.



                  For example, consider the statement: $$x > 2 ;land; x text{ is prime} implies x text{ is odd}.$$ Merely observing that this statement holds for some $x$ says very little — there are plenty of numbers for which it is trivially true, either because they are odd, or because they are not primes greater than 2. What makes this statement useful is that we can prove that it holds for all $x$ — there isn't a single number which would be greater than 2 and prime, but not odd.







                  share|cite|improve this answer














                  share|cite|improve this answer



                  share|cite|improve this answer








                  edited Mar 30 '15 at 12:10

























                  answered Sep 27 '14 at 14:23









                  Ilmari KaronenIlmari Karonen

                  19.6k25183




                  19.6k25183












                  • $begingroup$
                    This is the answer that gets to the heart of the matter. +1.
                    $endgroup$
                    – 6005
                    Sep 27 '14 at 16:18


















                  • $begingroup$
                    This is the answer that gets to the heart of the matter. +1.
                    $endgroup$
                    – 6005
                    Sep 27 '14 at 16:18
















                  $begingroup$
                  This is the answer that gets to the heart of the matter. +1.
                  $endgroup$
                  – 6005
                  Sep 27 '14 at 16:18




                  $begingroup$
                  This is the answer that gets to the heart of the matter. +1.
                  $endgroup$
                  – 6005
                  Sep 27 '14 at 16:18











                  11












                  $begingroup$

                  One way to understand implication is to remember that $ARightarrow B$ is equivalent to $neg A lor B$. If you understand negation ($neg$) and disjunction ($lor$), then you understand implication.






                  share|cite|improve this answer









                  $endgroup$









                  • 7




                    $begingroup$
                    I thought this was a strange equivalence, until I noticed myself saying things like: "I didn't know that or I would have done this"
                    $endgroup$
                    – Justin
                    Sep 28 '14 at 4:47








                  • 1




                    $begingroup$
                    I find $neg[Aland neg B]$ easier to work with.
                    $endgroup$
                    – Dan Christensen
                    Sep 29 '14 at 15:39
















                  11












                  $begingroup$

                  One way to understand implication is to remember that $ARightarrow B$ is equivalent to $neg A lor B$. If you understand negation ($neg$) and disjunction ($lor$), then you understand implication.






                  share|cite|improve this answer









                  $endgroup$









                  • 7




                    $begingroup$
                    I thought this was a strange equivalence, until I noticed myself saying things like: "I didn't know that or I would have done this"
                    $endgroup$
                    – Justin
                    Sep 28 '14 at 4:47








                  • 1




                    $begingroup$
                    I find $neg[Aland neg B]$ easier to work with.
                    $endgroup$
                    – Dan Christensen
                    Sep 29 '14 at 15:39














                  11












                  11








                  11





                  $begingroup$

                  One way to understand implication is to remember that $ARightarrow B$ is equivalent to $neg A lor B$. If you understand negation ($neg$) and disjunction ($lor$), then you understand implication.






                  share|cite|improve this answer









                  $endgroup$



                  One way to understand implication is to remember that $ARightarrow B$ is equivalent to $neg A lor B$. If you understand negation ($neg$) and disjunction ($lor$), then you understand implication.







                  share|cite|improve this answer












                  share|cite|improve this answer



                  share|cite|improve this answer










                  answered Sep 27 '14 at 20:30









                  Theodore NorvellTheodore Norvell

                  39119




                  39119








                  • 7




                    $begingroup$
                    I thought this was a strange equivalence, until I noticed myself saying things like: "I didn't know that or I would have done this"
                    $endgroup$
                    – Justin
                    Sep 28 '14 at 4:47








                  • 1




                    $begingroup$
                    I find $neg[Aland neg B]$ easier to work with.
                    $endgroup$
                    – Dan Christensen
                    Sep 29 '14 at 15:39














                  • 7




                    $begingroup$
                    I thought this was a strange equivalence, until I noticed myself saying things like: "I didn't know that or I would have done this"
                    $endgroup$
                    – Justin
                    Sep 28 '14 at 4:47








                  • 1




                    $begingroup$
                    I find $neg[Aland neg B]$ easier to work with.
                    $endgroup$
                    – Dan Christensen
                    Sep 29 '14 at 15:39








                  7




                  7




                  $begingroup$
                  I thought this was a strange equivalence, until I noticed myself saying things like: "I didn't know that or I would have done this"
                  $endgroup$
                  – Justin
                  Sep 28 '14 at 4:47






                  $begingroup$
                  I thought this was a strange equivalence, until I noticed myself saying things like: "I didn't know that or I would have done this"
                  $endgroup$
                  – Justin
                  Sep 28 '14 at 4:47






                  1




                  1




                  $begingroup$
                  I find $neg[Aland neg B]$ easier to work with.
                  $endgroup$
                  – Dan Christensen
                  Sep 29 '14 at 15:39




                  $begingroup$
                  I find $neg[Aland neg B]$ easier to work with.
                  $endgroup$
                  – Dan Christensen
                  Sep 29 '14 at 15:39











                  5












                  $begingroup$

                  Look at $A$ and $B$ as something that is either false or true. For
                  example let $A$ be the event that tomorrow is Tuesday and let $B$
                  be the event that the day after tomorrow is Wednesday.



                  Look at
                  $$
                  Aimplies B
                  $$



                  as a promise - if $A$ is true then so is $B$.



                  In our example, if $A$ is true then indeed so is $B$ and so the
                  implication $Aimplies B$ is true.



                  However, now consider $C$ as the statement that tomorrow is Friday,
                  and I state
                  $$
                  Cimplies B
                  $$



                  that is - I promise you that if $C$ will happen so will $B$.



                  Tomorrow is not Friday (at the time of writing), and so $C$ is false,
                  regardless of if $B$ is false or true - my promise was kept.



                  Now regarding the terminology of proofing $B$ is both $A,B$ are
                  true. Note that statements like
                  $$
                  text{My cat walks on four}implies1+1=2
                  $$



                  is true, since both are true, but what would it mean to prove $B$ from $A$
                  ?






                  share|cite|improve this answer











                  $endgroup$


















                    5












                    $begingroup$

                    Look at $A$ and $B$ as something that is either false or true. For
                    example let $A$ be the event that tomorrow is Tuesday and let $B$
                    be the event that the day after tomorrow is Wednesday.



                    Look at
                    $$
                    Aimplies B
                    $$



                    as a promise - if $A$ is true then so is $B$.



                    In our example, if $A$ is true then indeed so is $B$ and so the
                    implication $Aimplies B$ is true.



                    However, now consider $C$ as the statement that tomorrow is Friday,
                    and I state
                    $$
                    Cimplies B
                    $$



                    that is - I promise you that if $C$ will happen so will $B$.



                    Tomorrow is not Friday (at the time of writing), and so $C$ is false,
                    regardless of if $B$ is false or true - my promise was kept.



                    Now regarding the terminology of proofing $B$ is both $A,B$ are
                    true. Note that statements like
                    $$
                    text{My cat walks on four}implies1+1=2
                    $$



                    is true, since both are true, but what would it mean to prove $B$ from $A$
                    ?






                    share|cite|improve this answer











                    $endgroup$
















                      5












                      5








                      5





                      $begingroup$

                      Look at $A$ and $B$ as something that is either false or true. For
                      example let $A$ be the event that tomorrow is Tuesday and let $B$
                      be the event that the day after tomorrow is Wednesday.



                      Look at
                      $$
                      Aimplies B
                      $$



                      as a promise - if $A$ is true then so is $B$.



                      In our example, if $A$ is true then indeed so is $B$ and so the
                      implication $Aimplies B$ is true.



                      However, now consider $C$ as the statement that tomorrow is Friday,
                      and I state
                      $$
                      Cimplies B
                      $$



                      that is - I promise you that if $C$ will happen so will $B$.



                      Tomorrow is not Friday (at the time of writing), and so $C$ is false,
                      regardless of if $B$ is false or true - my promise was kept.



                      Now regarding the terminology of proofing $B$ is both $A,B$ are
                      true. Note that statements like
                      $$
                      text{My cat walks on four}implies1+1=2
                      $$



                      is true, since both are true, but what would it mean to prove $B$ from $A$
                      ?






                      share|cite|improve this answer











                      $endgroup$



                      Look at $A$ and $B$ as something that is either false or true. For
                      example let $A$ be the event that tomorrow is Tuesday and let $B$
                      be the event that the day after tomorrow is Wednesday.



                      Look at
                      $$
                      Aimplies B
                      $$



                      as a promise - if $A$ is true then so is $B$.



                      In our example, if $A$ is true then indeed so is $B$ and so the
                      implication $Aimplies B$ is true.



                      However, now consider $C$ as the statement that tomorrow is Friday,
                      and I state
                      $$
                      Cimplies B
                      $$



                      that is - I promise you that if $C$ will happen so will $B$.



                      Tomorrow is not Friday (at the time of writing), and so $C$ is false,
                      regardless of if $B$ is false or true - my promise was kept.



                      Now regarding the terminology of proofing $B$ is both $A,B$ are
                      true. Note that statements like
                      $$
                      text{My cat walks on four}implies1+1=2
                      $$



                      is true, since both are true, but what would it mean to prove $B$ from $A$
                      ?







                      share|cite|improve this answer














                      share|cite|improve this answer



                      share|cite|improve this answer








                      edited Sep 27 '14 at 10:24

























                      answered Sep 27 '14 at 10:18









                      BelgiBelgi

                      14.5k954113




                      14.5k954113























                          2












                          $begingroup$

                          I would say that $A$ being true and $B$ being true does not mean you can always prove (deduce) $B$ from $A$.



                          Here's an example. A: Alice lives in Atlanta. B: Bob lives in Boston. Even if these are both true, there is no (apparent) relationship. So you can't logically deduce $B$ from $A$ even though $ARightarrow B$ is true in this case.



                          I guess this means that if you can logically deduce statement $Q$ from statement $P$, then $PRightarrow Q$ is true; but knowing $PRightarrow Q$ is true does not guarantee the existence of a deduction of $Q$ from the assumption of $P$.



                          Logical implication is a defined logical connective, so as long as $P$ and $Q$ have truth values (true or false), so does $PRightarrow Q$.






                          share|cite|improve this answer









                          $endgroup$













                          • $begingroup$
                            You can logically deduce B from A in that example. Let's assume A is true. B is true. QED. Yes, this counts, unless you really meant A implies B and also not not-A implies not-B. But if you meant that, you should have said so.
                            $endgroup$
                            – djechlin
                            Sep 28 '14 at 2:00










                          • $begingroup$
                            Your 3rd paragraph is incorrect. Suppose $PRightarrow Q$ is given, we construct a proof of $Q$ to $P$: Assume $P$, assume $neg Q$, then by the truth table for $PRightarrow Q$ (or by modes ponens) $Q$. Hence $Q$ and $neg Q$, a contradiction. Hence our assumption $neg Q$ was false.
                            $endgroup$
                            – James
                            Sep 29 '14 at 16:41










                          • $begingroup$
                            My point is that sometimes two statements, $P$ and $Q$ are unrelated. Now $PRightarrow Q$ can perfectly well be true (provided $P$ is false or $Q$ is true), and yet knowing that $P$ is true does not help us determine (deduce) that $Q$ is true, because the statements are unrelated.
                            $endgroup$
                            – paw88789
                            Sep 29 '14 at 17:43










                          • $begingroup$
                            @paw88789 But when you know $P$, then to quote you "knowing $PRightarrow Q$ is true does not guarantee the existence of a deduction of $Q$ from the assumption that $P$." This is wrong, I just gave a deduction.
                            $endgroup$
                            – James
                            Sep 29 '14 at 18:42
















                          2












                          $begingroup$

                          I would say that $A$ being true and $B$ being true does not mean you can always prove (deduce) $B$ from $A$.



                          Here's an example. A: Alice lives in Atlanta. B: Bob lives in Boston. Even if these are both true, there is no (apparent) relationship. So you can't logically deduce $B$ from $A$ even though $ARightarrow B$ is true in this case.



                          I guess this means that if you can logically deduce statement $Q$ from statement $P$, then $PRightarrow Q$ is true; but knowing $PRightarrow Q$ is true does not guarantee the existence of a deduction of $Q$ from the assumption of $P$.



                          Logical implication is a defined logical connective, so as long as $P$ and $Q$ have truth values (true or false), so does $PRightarrow Q$.






                          share|cite|improve this answer









                          $endgroup$













                          • $begingroup$
                            You can logically deduce B from A in that example. Let's assume A is true. B is true. QED. Yes, this counts, unless you really meant A implies B and also not not-A implies not-B. But if you meant that, you should have said so.
                            $endgroup$
                            – djechlin
                            Sep 28 '14 at 2:00










                          • $begingroup$
                            Your 3rd paragraph is incorrect. Suppose $PRightarrow Q$ is given, we construct a proof of $Q$ to $P$: Assume $P$, assume $neg Q$, then by the truth table for $PRightarrow Q$ (or by modes ponens) $Q$. Hence $Q$ and $neg Q$, a contradiction. Hence our assumption $neg Q$ was false.
                            $endgroup$
                            – James
                            Sep 29 '14 at 16:41










                          • $begingroup$
                            My point is that sometimes two statements, $P$ and $Q$ are unrelated. Now $PRightarrow Q$ can perfectly well be true (provided $P$ is false or $Q$ is true), and yet knowing that $P$ is true does not help us determine (deduce) that $Q$ is true, because the statements are unrelated.
                            $endgroup$
                            – paw88789
                            Sep 29 '14 at 17:43










                          • $begingroup$
                            @paw88789 But when you know $P$, then to quote you "knowing $PRightarrow Q$ is true does not guarantee the existence of a deduction of $Q$ from the assumption that $P$." This is wrong, I just gave a deduction.
                            $endgroup$
                            – James
                            Sep 29 '14 at 18:42














                          2












                          2








                          2





                          $begingroup$

                          I would say that $A$ being true and $B$ being true does not mean you can always prove (deduce) $B$ from $A$.



                          Here's an example. A: Alice lives in Atlanta. B: Bob lives in Boston. Even if these are both true, there is no (apparent) relationship. So you can't logically deduce $B$ from $A$ even though $ARightarrow B$ is true in this case.



                          I guess this means that if you can logically deduce statement $Q$ from statement $P$, then $PRightarrow Q$ is true; but knowing $PRightarrow Q$ is true does not guarantee the existence of a deduction of $Q$ from the assumption of $P$.



                          Logical implication is a defined logical connective, so as long as $P$ and $Q$ have truth values (true or false), so does $PRightarrow Q$.






                          share|cite|improve this answer









                          $endgroup$



                          I would say that $A$ being true and $B$ being true does not mean you can always prove (deduce) $B$ from $A$.



                          Here's an example. A: Alice lives in Atlanta. B: Bob lives in Boston. Even if these are both true, there is no (apparent) relationship. So you can't logically deduce $B$ from $A$ even though $ARightarrow B$ is true in this case.



                          I guess this means that if you can logically deduce statement $Q$ from statement $P$, then $PRightarrow Q$ is true; but knowing $PRightarrow Q$ is true does not guarantee the existence of a deduction of $Q$ from the assumption of $P$.



                          Logical implication is a defined logical connective, so as long as $P$ and $Q$ have truth values (true or false), so does $PRightarrow Q$.







                          share|cite|improve this answer












                          share|cite|improve this answer



                          share|cite|improve this answer










                          answered Sep 27 '14 at 10:17









                          paw88789paw88789

                          29.1k12349




                          29.1k12349












                          • $begingroup$
                            You can logically deduce B from A in that example. Let's assume A is true. B is true. QED. Yes, this counts, unless you really meant A implies B and also not not-A implies not-B. But if you meant that, you should have said so.
                            $endgroup$
                            – djechlin
                            Sep 28 '14 at 2:00










                          • $begingroup$
                            Your 3rd paragraph is incorrect. Suppose $PRightarrow Q$ is given, we construct a proof of $Q$ to $P$: Assume $P$, assume $neg Q$, then by the truth table for $PRightarrow Q$ (or by modes ponens) $Q$. Hence $Q$ and $neg Q$, a contradiction. Hence our assumption $neg Q$ was false.
                            $endgroup$
                            – James
                            Sep 29 '14 at 16:41










                          • $begingroup$
                            My point is that sometimes two statements, $P$ and $Q$ are unrelated. Now $PRightarrow Q$ can perfectly well be true (provided $P$ is false or $Q$ is true), and yet knowing that $P$ is true does not help us determine (deduce) that $Q$ is true, because the statements are unrelated.
                            $endgroup$
                            – paw88789
                            Sep 29 '14 at 17:43










                          • $begingroup$
                            @paw88789 But when you know $P$, then to quote you "knowing $PRightarrow Q$ is true does not guarantee the existence of a deduction of $Q$ from the assumption that $P$." This is wrong, I just gave a deduction.
                            $endgroup$
                            – James
                            Sep 29 '14 at 18:42


















                          • $begingroup$
                            You can logically deduce B from A in that example. Let's assume A is true. B is true. QED. Yes, this counts, unless you really meant A implies B and also not not-A implies not-B. But if you meant that, you should have said so.
                            $endgroup$
                            – djechlin
                            Sep 28 '14 at 2:00










                          • $begingroup$
                            Your 3rd paragraph is incorrect. Suppose $PRightarrow Q$ is given, we construct a proof of $Q$ to $P$: Assume $P$, assume $neg Q$, then by the truth table for $PRightarrow Q$ (or by modes ponens) $Q$. Hence $Q$ and $neg Q$, a contradiction. Hence our assumption $neg Q$ was false.
                            $endgroup$
                            – James
                            Sep 29 '14 at 16:41










                          • $begingroup$
                            My point is that sometimes two statements, $P$ and $Q$ are unrelated. Now $PRightarrow Q$ can perfectly well be true (provided $P$ is false or $Q$ is true), and yet knowing that $P$ is true does not help us determine (deduce) that $Q$ is true, because the statements are unrelated.
                            $endgroup$
                            – paw88789
                            Sep 29 '14 at 17:43










                          • $begingroup$
                            @paw88789 But when you know $P$, then to quote you "knowing $PRightarrow Q$ is true does not guarantee the existence of a deduction of $Q$ from the assumption that $P$." This is wrong, I just gave a deduction.
                            $endgroup$
                            – James
                            Sep 29 '14 at 18:42
















                          $begingroup$
                          You can logically deduce B from A in that example. Let's assume A is true. B is true. QED. Yes, this counts, unless you really meant A implies B and also not not-A implies not-B. But if you meant that, you should have said so.
                          $endgroup$
                          – djechlin
                          Sep 28 '14 at 2:00




                          $begingroup$
                          You can logically deduce B from A in that example. Let's assume A is true. B is true. QED. Yes, this counts, unless you really meant A implies B and also not not-A implies not-B. But if you meant that, you should have said so.
                          $endgroup$
                          – djechlin
                          Sep 28 '14 at 2:00












                          $begingroup$
                          Your 3rd paragraph is incorrect. Suppose $PRightarrow Q$ is given, we construct a proof of $Q$ to $P$: Assume $P$, assume $neg Q$, then by the truth table for $PRightarrow Q$ (or by modes ponens) $Q$. Hence $Q$ and $neg Q$, a contradiction. Hence our assumption $neg Q$ was false.
                          $endgroup$
                          – James
                          Sep 29 '14 at 16:41




                          $begingroup$
                          Your 3rd paragraph is incorrect. Suppose $PRightarrow Q$ is given, we construct a proof of $Q$ to $P$: Assume $P$, assume $neg Q$, then by the truth table for $PRightarrow Q$ (or by modes ponens) $Q$. Hence $Q$ and $neg Q$, a contradiction. Hence our assumption $neg Q$ was false.
                          $endgroup$
                          – James
                          Sep 29 '14 at 16:41












                          $begingroup$
                          My point is that sometimes two statements, $P$ and $Q$ are unrelated. Now $PRightarrow Q$ can perfectly well be true (provided $P$ is false or $Q$ is true), and yet knowing that $P$ is true does not help us determine (deduce) that $Q$ is true, because the statements are unrelated.
                          $endgroup$
                          – paw88789
                          Sep 29 '14 at 17:43




                          $begingroup$
                          My point is that sometimes two statements, $P$ and $Q$ are unrelated. Now $PRightarrow Q$ can perfectly well be true (provided $P$ is false or $Q$ is true), and yet knowing that $P$ is true does not help us determine (deduce) that $Q$ is true, because the statements are unrelated.
                          $endgroup$
                          – paw88789
                          Sep 29 '14 at 17:43












                          $begingroup$
                          @paw88789 But when you know $P$, then to quote you "knowing $PRightarrow Q$ is true does not guarantee the existence of a deduction of $Q$ from the assumption that $P$." This is wrong, I just gave a deduction.
                          $endgroup$
                          – James
                          Sep 29 '14 at 18:42




                          $begingroup$
                          @paw88789 But when you know $P$, then to quote you "knowing $PRightarrow Q$ is true does not guarantee the existence of a deduction of $Q$ from the assumption that $P$." This is wrong, I just gave a deduction.
                          $endgroup$
                          – James
                          Sep 29 '14 at 18:42











                          2












                          $begingroup$

                          Maybe it's more clear if we separate the logical operator meaning of implication from its logical statement meaning.



                          When we use it as a logical operator, we conceive it simply as an entity, that given two logical values (thus true or false), produces a third logical value, using a common defined rule (its truth table).

                          So it makes perfect sense to say $A implies B = true$ if $A = B = true$, and we don't concern about what actually proposition $A$ and $B$ means, we care exclusively about their logical values.



                          It's different when we use implication as a logical statement.

                          In this case we really say something about the meaning of the propositions involved in our statement. So, while proposition $A = My;cat;is;black$ is true, and proposition $B = I;am;hungry$ is true as well, $A implies B$ is not a valid statement.

                          Such meaning is linked to set theory and formal logic. Using implication in this context means that you can infer $B$ from $A$, in a way called modus ponens.






                          share|cite|improve this answer











                          $endgroup$


















                            2












                            $begingroup$

                            Maybe it's more clear if we separate the logical operator meaning of implication from its logical statement meaning.



                            When we use it as a logical operator, we conceive it simply as an entity, that given two logical values (thus true or false), produces a third logical value, using a common defined rule (its truth table).

                            So it makes perfect sense to say $A implies B = true$ if $A = B = true$, and we don't concern about what actually proposition $A$ and $B$ means, we care exclusively about their logical values.



                            It's different when we use implication as a logical statement.

                            In this case we really say something about the meaning of the propositions involved in our statement. So, while proposition $A = My;cat;is;black$ is true, and proposition $B = I;am;hungry$ is true as well, $A implies B$ is not a valid statement.

                            Such meaning is linked to set theory and formal logic. Using implication in this context means that you can infer $B$ from $A$, in a way called modus ponens.






                            share|cite|improve this answer











                            $endgroup$
















                              2












                              2








                              2





                              $begingroup$

                              Maybe it's more clear if we separate the logical operator meaning of implication from its logical statement meaning.



                              When we use it as a logical operator, we conceive it simply as an entity, that given two logical values (thus true or false), produces a third logical value, using a common defined rule (its truth table).

                              So it makes perfect sense to say $A implies B = true$ if $A = B = true$, and we don't concern about what actually proposition $A$ and $B$ means, we care exclusively about their logical values.



                              It's different when we use implication as a logical statement.

                              In this case we really say something about the meaning of the propositions involved in our statement. So, while proposition $A = My;cat;is;black$ is true, and proposition $B = I;am;hungry$ is true as well, $A implies B$ is not a valid statement.

                              Such meaning is linked to set theory and formal logic. Using implication in this context means that you can infer $B$ from $A$, in a way called modus ponens.






                              share|cite|improve this answer











                              $endgroup$



                              Maybe it's more clear if we separate the logical operator meaning of implication from its logical statement meaning.



                              When we use it as a logical operator, we conceive it simply as an entity, that given two logical values (thus true or false), produces a third logical value, using a common defined rule (its truth table).

                              So it makes perfect sense to say $A implies B = true$ if $A = B = true$, and we don't concern about what actually proposition $A$ and $B$ means, we care exclusively about their logical values.



                              It's different when we use implication as a logical statement.

                              In this case we really say something about the meaning of the propositions involved in our statement. So, while proposition $A = My;cat;is;black$ is true, and proposition $B = I;am;hungry$ is true as well, $A implies B$ is not a valid statement.

                              Such meaning is linked to set theory and formal logic. Using implication in this context means that you can infer $B$ from $A$, in a way called modus ponens.







                              share|cite|improve this answer














                              share|cite|improve this answer



                              share|cite|improve this answer








                              edited Sep 27 '14 at 11:31

























                              answered Sep 27 '14 at 11:11









                              mattecapumattecapu

                              695618




                              695618























                                  2












                                  $begingroup$

                                  Implication can be understood as thinking in NECESSARY and SUFFICIENT conditions. A good example is to think about someone who has born in Dallas, Texas.



                                  Proposition P could stand for "Someone who was born in Dallas"



                                  Proposition Q could stand for "Someone who is texan"



                                  Truth table for implication is:



                                  enter image description here



                                  So, in this example is easy to check line by line the validity of truth table above:




                                  • Who was born in Dallas is therefore texan; So the first implication is TRUE.


                                  • Who was born in Dallas MUST be texan. So, saying that someone who was born in Dallas, in the state of Texas, and is not texan is FALSE;


                                  • Saying who wasn't born in Dallas but is texan anyway (take someone who has born in Houston, for example) is a TRUE statement;


                                  • At last, who wasn't born in Dallas AND also is NOT texan is also TRUE.



                                  So, thinking in terms of NECESSARY (to be texan is a necessary condition for who was born in Dallas) and SUFFICIENT (to born in Dallas is a sufficient condition to be texan) conditions ease the process to translate implication to natural language.



                                  Other examples of propositions containing necessary and sufficient conditions are:




                                  • P = To be a priest / Q = To believe in God;

                                  • P = To be a Police officer / Q = To carry a gun;

                                  • P = The number is divisible by 4 / Q = The number is even.






                                  share|cite|improve this answer











                                  $endgroup$


















                                    2












                                    $begingroup$

                                    Implication can be understood as thinking in NECESSARY and SUFFICIENT conditions. A good example is to think about someone who has born in Dallas, Texas.



                                    Proposition P could stand for "Someone who was born in Dallas"



                                    Proposition Q could stand for "Someone who is texan"



                                    Truth table for implication is:



                                    enter image description here



                                    So, in this example is easy to check line by line the validity of truth table above:




                                    • Who was born in Dallas is therefore texan; So the first implication is TRUE.


                                    • Who was born in Dallas MUST be texan. So, saying that someone who was born in Dallas, in the state of Texas, and is not texan is FALSE;


                                    • Saying who wasn't born in Dallas but is texan anyway (take someone who has born in Houston, for example) is a TRUE statement;


                                    • At last, who wasn't born in Dallas AND also is NOT texan is also TRUE.



                                    So, thinking in terms of NECESSARY (to be texan is a necessary condition for who was born in Dallas) and SUFFICIENT (to born in Dallas is a sufficient condition to be texan) conditions ease the process to translate implication to natural language.



                                    Other examples of propositions containing necessary and sufficient conditions are:




                                    • P = To be a priest / Q = To believe in God;

                                    • P = To be a Police officer / Q = To carry a gun;

                                    • P = The number is divisible by 4 / Q = The number is even.






                                    share|cite|improve this answer











                                    $endgroup$
















                                      2












                                      2








                                      2





                                      $begingroup$

                                      Implication can be understood as thinking in NECESSARY and SUFFICIENT conditions. A good example is to think about someone who has born in Dallas, Texas.



                                      Proposition P could stand for "Someone who was born in Dallas"



                                      Proposition Q could stand for "Someone who is texan"



                                      Truth table for implication is:



                                      enter image description here



                                      So, in this example is easy to check line by line the validity of truth table above:




                                      • Who was born in Dallas is therefore texan; So the first implication is TRUE.


                                      • Who was born in Dallas MUST be texan. So, saying that someone who was born in Dallas, in the state of Texas, and is not texan is FALSE;


                                      • Saying who wasn't born in Dallas but is texan anyway (take someone who has born in Houston, for example) is a TRUE statement;


                                      • At last, who wasn't born in Dallas AND also is NOT texan is also TRUE.



                                      So, thinking in terms of NECESSARY (to be texan is a necessary condition for who was born in Dallas) and SUFFICIENT (to born in Dallas is a sufficient condition to be texan) conditions ease the process to translate implication to natural language.



                                      Other examples of propositions containing necessary and sufficient conditions are:




                                      • P = To be a priest / Q = To believe in God;

                                      • P = To be a Police officer / Q = To carry a gun;

                                      • P = The number is divisible by 4 / Q = The number is even.






                                      share|cite|improve this answer











                                      $endgroup$



                                      Implication can be understood as thinking in NECESSARY and SUFFICIENT conditions. A good example is to think about someone who has born in Dallas, Texas.



                                      Proposition P could stand for "Someone who was born in Dallas"



                                      Proposition Q could stand for "Someone who is texan"



                                      Truth table for implication is:



                                      enter image description here



                                      So, in this example is easy to check line by line the validity of truth table above:




                                      • Who was born in Dallas is therefore texan; So the first implication is TRUE.


                                      • Who was born in Dallas MUST be texan. So, saying that someone who was born in Dallas, in the state of Texas, and is not texan is FALSE;


                                      • Saying who wasn't born in Dallas but is texan anyway (take someone who has born in Houston, for example) is a TRUE statement;


                                      • At last, who wasn't born in Dallas AND also is NOT texan is also TRUE.



                                      So, thinking in terms of NECESSARY (to be texan is a necessary condition for who was born in Dallas) and SUFFICIENT (to born in Dallas is a sufficient condition to be texan) conditions ease the process to translate implication to natural language.



                                      Other examples of propositions containing necessary and sufficient conditions are:




                                      • P = To be a priest / Q = To believe in God;

                                      • P = To be a Police officer / Q = To carry a gun;

                                      • P = The number is divisible by 4 / Q = The number is even.







                                      share|cite|improve this answer














                                      share|cite|improve this answer



                                      share|cite|improve this answer








                                      edited Mar 30 '15 at 16:19

























                                      answered Mar 30 '15 at 9:37









                                      Daniel LavieriDaniel Lavieri

                                      213




                                      213























                                          1












                                          $begingroup$

                                          The problem is understanding that there's a difference between the everyday mathematical meaning of "implies" - "I can prove it (or someone can)" - and the formal logical meaning expressed by the truth table. This may help: In classical logic, why is $(pRightarrow q)$ True if $p$ is False and $q$ is True?.






                                          share|cite|improve this answer











                                          $endgroup$


















                                            1












                                            $begingroup$

                                            The problem is understanding that there's a difference between the everyday mathematical meaning of "implies" - "I can prove it (or someone can)" - and the formal logical meaning expressed by the truth table. This may help: In classical logic, why is $(pRightarrow q)$ True if $p$ is False and $q$ is True?.






                                            share|cite|improve this answer











                                            $endgroup$
















                                              1












                                              1








                                              1





                                              $begingroup$

                                              The problem is understanding that there's a difference between the everyday mathematical meaning of "implies" - "I can prove it (or someone can)" - and the formal logical meaning expressed by the truth table. This may help: In classical logic, why is $(pRightarrow q)$ True if $p$ is False and $q$ is True?.






                                              share|cite|improve this answer











                                              $endgroup$



                                              The problem is understanding that there's a difference between the everyday mathematical meaning of "implies" - "I can prove it (or someone can)" - and the formal logical meaning expressed by the truth table. This may help: In classical logic, why is $(pRightarrow q)$ True if $p$ is False and $q$ is True?.







                                              share|cite|improve this answer














                                              share|cite|improve this answer



                                              share|cite|improve this answer








                                              edited Apr 13 '17 at 12:21









                                              Community

                                              1




                                              1










                                              answered Sep 27 '14 at 22:22









                                              Ethan BolkerEthan Bolker

                                              42.1k548111




                                              42.1k548111























                                                  0












                                                  $begingroup$


                                                  If sentence $A$ is true and $B$ is true then $Aimplies B$ is true.




                                                  Yes. In mathematics and logic as widely applied, $Aimplies B$ is equivalent to $neg[Alandneg B]$.



                                                  So, if $A$ is true and $B$ is true, then $neg[Alandneg B]$ and hence $Aimplies B$ will also be true.




                                                  Does that mean if $A$ and $B$ are both true then there is a way to prove $B$ is true from $A$, always?




                                                  Yes. Here is an informal proof:




                                                  1. Suppose $A land B$.


                                                  2. Suppose $A$.


                                                  3. $B$ (from 1)


                                                  4. $Aimplies B$ (conclusion from 2 and 3)


                                                  5. $Aland B implies [Aimplies B]$ (conclusion from 1 and 4)



                                                  Alternatively, prove (5) using truth table.




                                                  The same for if $A$ is false can you get anything either True or false proved from this $A$?




                                                  Yes. Informal proof:




                                                  1. Suppose $neg A$


                                                  2. Suppose $Aland neg B$


                                                  3. $Aland neg A$ (contradiction from 1 and 2)


                                                  4. $neg[Aland neg B]$ (by contradiciton from 2 and 3)


                                                  5. $Aimplies B$ from (equivalently from 4)


                                                  6. $neg Aimplies [Aimplies B] $ (conclusion from 1 and 5)



                                                  Alternatively, prove (6) using a truth table.





                                                  That the above may seem counter-intuitive may stem from the fact that implication is often confused with temporal notions of causality. The statement, "If it is raining, then it is cloudy" on its own does not mean that rain always causes clouds, or that clouds always cause rain. If means only that, at a given instant in time, its is not the case that it is raining and not cloudy.






                                                  share|cite|improve this answer











                                                  $endgroup$


















                                                    0












                                                    $begingroup$


                                                    If sentence $A$ is true and $B$ is true then $Aimplies B$ is true.




                                                    Yes. In mathematics and logic as widely applied, $Aimplies B$ is equivalent to $neg[Alandneg B]$.



                                                    So, if $A$ is true and $B$ is true, then $neg[Alandneg B]$ and hence $Aimplies B$ will also be true.




                                                    Does that mean if $A$ and $B$ are both true then there is a way to prove $B$ is true from $A$, always?




                                                    Yes. Here is an informal proof:




                                                    1. Suppose $A land B$.


                                                    2. Suppose $A$.


                                                    3. $B$ (from 1)


                                                    4. $Aimplies B$ (conclusion from 2 and 3)


                                                    5. $Aland B implies [Aimplies B]$ (conclusion from 1 and 4)



                                                    Alternatively, prove (5) using truth table.




                                                    The same for if $A$ is false can you get anything either True or false proved from this $A$?




                                                    Yes. Informal proof:




                                                    1. Suppose $neg A$


                                                    2. Suppose $Aland neg B$


                                                    3. $Aland neg A$ (contradiction from 1 and 2)


                                                    4. $neg[Aland neg B]$ (by contradiciton from 2 and 3)


                                                    5. $Aimplies B$ from (equivalently from 4)


                                                    6. $neg Aimplies [Aimplies B] $ (conclusion from 1 and 5)



                                                    Alternatively, prove (6) using a truth table.





                                                    That the above may seem counter-intuitive may stem from the fact that implication is often confused with temporal notions of causality. The statement, "If it is raining, then it is cloudy" on its own does not mean that rain always causes clouds, or that clouds always cause rain. If means only that, at a given instant in time, its is not the case that it is raining and not cloudy.






                                                    share|cite|improve this answer











                                                    $endgroup$
















                                                      0












                                                      0








                                                      0





                                                      $begingroup$


                                                      If sentence $A$ is true and $B$ is true then $Aimplies B$ is true.




                                                      Yes. In mathematics and logic as widely applied, $Aimplies B$ is equivalent to $neg[Alandneg B]$.



                                                      So, if $A$ is true and $B$ is true, then $neg[Alandneg B]$ and hence $Aimplies B$ will also be true.




                                                      Does that mean if $A$ and $B$ are both true then there is a way to prove $B$ is true from $A$, always?




                                                      Yes. Here is an informal proof:




                                                      1. Suppose $A land B$.


                                                      2. Suppose $A$.


                                                      3. $B$ (from 1)


                                                      4. $Aimplies B$ (conclusion from 2 and 3)


                                                      5. $Aland B implies [Aimplies B]$ (conclusion from 1 and 4)



                                                      Alternatively, prove (5) using truth table.




                                                      The same for if $A$ is false can you get anything either True or false proved from this $A$?




                                                      Yes. Informal proof:




                                                      1. Suppose $neg A$


                                                      2. Suppose $Aland neg B$


                                                      3. $Aland neg A$ (contradiction from 1 and 2)


                                                      4. $neg[Aland neg B]$ (by contradiciton from 2 and 3)


                                                      5. $Aimplies B$ from (equivalently from 4)


                                                      6. $neg Aimplies [Aimplies B] $ (conclusion from 1 and 5)



                                                      Alternatively, prove (6) using a truth table.





                                                      That the above may seem counter-intuitive may stem from the fact that implication is often confused with temporal notions of causality. The statement, "If it is raining, then it is cloudy" on its own does not mean that rain always causes clouds, or that clouds always cause rain. If means only that, at a given instant in time, its is not the case that it is raining and not cloudy.






                                                      share|cite|improve this answer











                                                      $endgroup$




                                                      If sentence $A$ is true and $B$ is true then $Aimplies B$ is true.




                                                      Yes. In mathematics and logic as widely applied, $Aimplies B$ is equivalent to $neg[Alandneg B]$.



                                                      So, if $A$ is true and $B$ is true, then $neg[Alandneg B]$ and hence $Aimplies B$ will also be true.




                                                      Does that mean if $A$ and $B$ are both true then there is a way to prove $B$ is true from $A$, always?




                                                      Yes. Here is an informal proof:




                                                      1. Suppose $A land B$.


                                                      2. Suppose $A$.


                                                      3. $B$ (from 1)


                                                      4. $Aimplies B$ (conclusion from 2 and 3)


                                                      5. $Aland B implies [Aimplies B]$ (conclusion from 1 and 4)



                                                      Alternatively, prove (5) using truth table.




                                                      The same for if $A$ is false can you get anything either True or false proved from this $A$?




                                                      Yes. Informal proof:




                                                      1. Suppose $neg A$


                                                      2. Suppose $Aland neg B$


                                                      3. $Aland neg A$ (contradiction from 1 and 2)


                                                      4. $neg[Aland neg B]$ (by contradiciton from 2 and 3)


                                                      5. $Aimplies B$ from (equivalently from 4)


                                                      6. $neg Aimplies [Aimplies B] $ (conclusion from 1 and 5)



                                                      Alternatively, prove (6) using a truth table.





                                                      That the above may seem counter-intuitive may stem from the fact that implication is often confused with temporal notions of causality. The statement, "If it is raining, then it is cloudy" on its own does not mean that rain always causes clouds, or that clouds always cause rain. If means only that, at a given instant in time, its is not the case that it is raining and not cloudy.







                                                      share|cite|improve this answer














                                                      share|cite|improve this answer



                                                      share|cite|improve this answer








                                                      edited Sep 29 '14 at 18:49

























                                                      answered Sep 29 '14 at 15:55









                                                      Dan ChristensenDan Christensen

                                                      8,55321833




                                                      8,55321833























                                                          0












                                                          $begingroup$

                                                          The sentence $Aimplies B$ is not a sentence about proving anything. It is a compound sentence, but otherwise just like the sentences $A$ and $B$. For the truth table, the compound sentence got to have truth values like $A$ and $B$ has, and it has to depend on these sentences truth values. The values for this compound sentence is the same as for the compound statement $neg A vee B$.






                                                          share|cite|improve this answer









                                                          $endgroup$


















                                                            0












                                                            $begingroup$

                                                            The sentence $Aimplies B$ is not a sentence about proving anything. It is a compound sentence, but otherwise just like the sentences $A$ and $B$. For the truth table, the compound sentence got to have truth values like $A$ and $B$ has, and it has to depend on these sentences truth values. The values for this compound sentence is the same as for the compound statement $neg A vee B$.






                                                            share|cite|improve this answer









                                                            $endgroup$
















                                                              0












                                                              0








                                                              0





                                                              $begingroup$

                                                              The sentence $Aimplies B$ is not a sentence about proving anything. It is a compound sentence, but otherwise just like the sentences $A$ and $B$. For the truth table, the compound sentence got to have truth values like $A$ and $B$ has, and it has to depend on these sentences truth values. The values for this compound sentence is the same as for the compound statement $neg A vee B$.






                                                              share|cite|improve this answer









                                                              $endgroup$



                                                              The sentence $Aimplies B$ is not a sentence about proving anything. It is a compound sentence, but otherwise just like the sentences $A$ and $B$. For the truth table, the compound sentence got to have truth values like $A$ and $B$ has, and it has to depend on these sentences truth values. The values for this compound sentence is the same as for the compound statement $neg A vee B$.







                                                              share|cite|improve this answer












                                                              share|cite|improve this answer



                                                              share|cite|improve this answer










                                                              answered Sep 29 '14 at 19:14









                                                              LehsLehs

                                                              6,98031662




                                                              6,98031662























                                                                  0












                                                                  $begingroup$

                                                                  This is the way I understand it (I make no guarantee this is considered a correct interpretation).



                                                                  the truth value of an implies statement is basically 'innocent until proven guilty', or 'my statement is true so long as the proposition values don't disprove it'.



                                                                  So with an example, propositions P and Q, with the statement P implies Q. Lets say proposition P is 'is a man' and proposition Q is 'lives in japan', so our implication statement is basically saying 'all men live in japan'.



                                                                  The truth table:



                                                                  truth table



                                                                  is basically saying, for any example where P and Q are true, we haven't disproved our statement that P -> Q, and for any example where P is false we haven't disproved our statement, only if we have an example of P being true and Q being false is our statement shown to be false.



                                                                  So imagine there are only 4 humans in existence, we look at each of them and categorise them in our truth table. By doing this we can work out if our statement that P -> Q is true. We find 2 of the people are men, and that they do live in Japan, OK good, our statement still holds true, the 3rd person is a woman, we don't care if she lives in Japan or not because in either case our statement about men is still holding true.



                                                                  A woman living in or outside japan has no bearing on the truth of our statement about men living in japan.



                                                                  The 4th person then will decide if our statement is true or not, if the 4th person is a man living in japan, or a woman, then is it logically true to say that being a man implies that you live in japan.



                                                                  Why is this useful? well the truth table itself doesn't help us prove the truth of our statement by itself, what it does is clearly set out what conditions are relevant to the truth of our statement - i.e. that we don't care about women, and that only a man not living in japan will falsify our statement.



                                                                  Of course, you could make a statement like 'all green men live on mars'. This is a true statement because we cannot verify who lives on mars, and we don't have any examples of green men. But this is whats called a "vacuous truth", it has no bearing on reality for us because the propositions are intangible.






                                                                  share|cite|improve this answer









                                                                  $endgroup$


















                                                                    0












                                                                    $begingroup$

                                                                    This is the way I understand it (I make no guarantee this is considered a correct interpretation).



                                                                    the truth value of an implies statement is basically 'innocent until proven guilty', or 'my statement is true so long as the proposition values don't disprove it'.



                                                                    So with an example, propositions P and Q, with the statement P implies Q. Lets say proposition P is 'is a man' and proposition Q is 'lives in japan', so our implication statement is basically saying 'all men live in japan'.



                                                                    The truth table:



                                                                    truth table



                                                                    is basically saying, for any example where P and Q are true, we haven't disproved our statement that P -> Q, and for any example where P is false we haven't disproved our statement, only if we have an example of P being true and Q being false is our statement shown to be false.



                                                                    So imagine there are only 4 humans in existence, we look at each of them and categorise them in our truth table. By doing this we can work out if our statement that P -> Q is true. We find 2 of the people are men, and that they do live in Japan, OK good, our statement still holds true, the 3rd person is a woman, we don't care if she lives in Japan or not because in either case our statement about men is still holding true.



                                                                    A woman living in or outside japan has no bearing on the truth of our statement about men living in japan.



                                                                    The 4th person then will decide if our statement is true or not, if the 4th person is a man living in japan, or a woman, then is it logically true to say that being a man implies that you live in japan.



                                                                    Why is this useful? well the truth table itself doesn't help us prove the truth of our statement by itself, what it does is clearly set out what conditions are relevant to the truth of our statement - i.e. that we don't care about women, and that only a man not living in japan will falsify our statement.



                                                                    Of course, you could make a statement like 'all green men live on mars'. This is a true statement because we cannot verify who lives on mars, and we don't have any examples of green men. But this is whats called a "vacuous truth", it has no bearing on reality for us because the propositions are intangible.






                                                                    share|cite|improve this answer









                                                                    $endgroup$
















                                                                      0












                                                                      0








                                                                      0





                                                                      $begingroup$

                                                                      This is the way I understand it (I make no guarantee this is considered a correct interpretation).



                                                                      the truth value of an implies statement is basically 'innocent until proven guilty', or 'my statement is true so long as the proposition values don't disprove it'.



                                                                      So with an example, propositions P and Q, with the statement P implies Q. Lets say proposition P is 'is a man' and proposition Q is 'lives in japan', so our implication statement is basically saying 'all men live in japan'.



                                                                      The truth table:



                                                                      truth table



                                                                      is basically saying, for any example where P and Q are true, we haven't disproved our statement that P -> Q, and for any example where P is false we haven't disproved our statement, only if we have an example of P being true and Q being false is our statement shown to be false.



                                                                      So imagine there are only 4 humans in existence, we look at each of them and categorise them in our truth table. By doing this we can work out if our statement that P -> Q is true. We find 2 of the people are men, and that they do live in Japan, OK good, our statement still holds true, the 3rd person is a woman, we don't care if she lives in Japan or not because in either case our statement about men is still holding true.



                                                                      A woman living in or outside japan has no bearing on the truth of our statement about men living in japan.



                                                                      The 4th person then will decide if our statement is true or not, if the 4th person is a man living in japan, or a woman, then is it logically true to say that being a man implies that you live in japan.



                                                                      Why is this useful? well the truth table itself doesn't help us prove the truth of our statement by itself, what it does is clearly set out what conditions are relevant to the truth of our statement - i.e. that we don't care about women, and that only a man not living in japan will falsify our statement.



                                                                      Of course, you could make a statement like 'all green men live on mars'. This is a true statement because we cannot verify who lives on mars, and we don't have any examples of green men. But this is whats called a "vacuous truth", it has no bearing on reality for us because the propositions are intangible.






                                                                      share|cite|improve this answer









                                                                      $endgroup$



                                                                      This is the way I understand it (I make no guarantee this is considered a correct interpretation).



                                                                      the truth value of an implies statement is basically 'innocent until proven guilty', or 'my statement is true so long as the proposition values don't disprove it'.



                                                                      So with an example, propositions P and Q, with the statement P implies Q. Lets say proposition P is 'is a man' and proposition Q is 'lives in japan', so our implication statement is basically saying 'all men live in japan'.



                                                                      The truth table:



                                                                      truth table



                                                                      is basically saying, for any example where P and Q are true, we haven't disproved our statement that P -> Q, and for any example where P is false we haven't disproved our statement, only if we have an example of P being true and Q being false is our statement shown to be false.



                                                                      So imagine there are only 4 humans in existence, we look at each of them and categorise them in our truth table. By doing this we can work out if our statement that P -> Q is true. We find 2 of the people are men, and that they do live in Japan, OK good, our statement still holds true, the 3rd person is a woman, we don't care if she lives in Japan or not because in either case our statement about men is still holding true.



                                                                      A woman living in or outside japan has no bearing on the truth of our statement about men living in japan.



                                                                      The 4th person then will decide if our statement is true or not, if the 4th person is a man living in japan, or a woman, then is it logically true to say that being a man implies that you live in japan.



                                                                      Why is this useful? well the truth table itself doesn't help us prove the truth of our statement by itself, what it does is clearly set out what conditions are relevant to the truth of our statement - i.e. that we don't care about women, and that only a man not living in japan will falsify our statement.



                                                                      Of course, you could make a statement like 'all green men live on mars'. This is a true statement because we cannot verify who lives on mars, and we don't have any examples of green men. But this is whats called a "vacuous truth", it has no bearing on reality for us because the propositions are intangible.







                                                                      share|cite|improve this answer












                                                                      share|cite|improve this answer



                                                                      share|cite|improve this answer










                                                                      answered Nov 22 '17 at 9:13









                                                                      Coder375Coder375

                                                                      101




                                                                      101























                                                                          -2












                                                                          $begingroup$

                                                                          This illustration of logical implication might help:



                                                                          a) When you truly understand logical implication,

                                                                          b) then you’ll be a happy person.



                                                                          You may be a happy person for other reasons. But if you’re unhappy, then certainly you don’t truly understand logical implication. :-)



                                                                          Truth table:

                                                                          a b a⇒b

                                                                          F F T

                                                                          F T T

                                                                          T F F

                                                                          T T T



                                                                          I hope this somewhat intuitive example helps.






                                                                          share|cite|improve this answer











                                                                          $endgroup$


















                                                                            -2












                                                                            $begingroup$

                                                                            This illustration of logical implication might help:



                                                                            a) When you truly understand logical implication,

                                                                            b) then you’ll be a happy person.



                                                                            You may be a happy person for other reasons. But if you’re unhappy, then certainly you don’t truly understand logical implication. :-)



                                                                            Truth table:

                                                                            a b a⇒b

                                                                            F F T

                                                                            F T T

                                                                            T F F

                                                                            T T T



                                                                            I hope this somewhat intuitive example helps.






                                                                            share|cite|improve this answer











                                                                            $endgroup$
















                                                                              -2












                                                                              -2








                                                                              -2





                                                                              $begingroup$

                                                                              This illustration of logical implication might help:



                                                                              a) When you truly understand logical implication,

                                                                              b) then you’ll be a happy person.



                                                                              You may be a happy person for other reasons. But if you’re unhappy, then certainly you don’t truly understand logical implication. :-)



                                                                              Truth table:

                                                                              a b a⇒b

                                                                              F F T

                                                                              F T T

                                                                              T F F

                                                                              T T T



                                                                              I hope this somewhat intuitive example helps.






                                                                              share|cite|improve this answer











                                                                              $endgroup$



                                                                              This illustration of logical implication might help:



                                                                              a) When you truly understand logical implication,

                                                                              b) then you’ll be a happy person.



                                                                              You may be a happy person for other reasons. But if you’re unhappy, then certainly you don’t truly understand logical implication. :-)



                                                                              Truth table:

                                                                              a b a⇒b

                                                                              F F T

                                                                              F T T

                                                                              T F F

                                                                              T T T



                                                                              I hope this somewhat intuitive example helps.







                                                                              share|cite|improve this answer














                                                                              share|cite|improve this answer



                                                                              share|cite|improve this answer








                                                                              edited Jan 6 at 9:41

























                                                                              answered Jan 6 at 5:52









                                                                              Bert SierraBert Sierra

                                                                              12




                                                                              12






























                                                                                  draft saved

                                                                                  draft discarded




















































                                                                                  Thanks for contributing an answer to Mathematics Stack Exchange!


                                                                                  • Please be sure to answer the question. Provide details and share your research!

                                                                                  But avoid



                                                                                  • Asking for help, clarification, or responding to other answers.

                                                                                  • Making statements based on opinion; back them up with references or personal experience.


                                                                                  Use MathJax to format equations. MathJax reference.


                                                                                  To learn more, see our tips on writing great answers.




                                                                                  draft saved


                                                                                  draft discarded














                                                                                  StackExchange.ready(
                                                                                  function () {
                                                                                  StackExchange.openid.initPostLogin('.new-post-login', 'https%3a%2f%2fmath.stackexchange.com%2fquestions%2f947989%2fi-cant-understand-logical-implication%23new-answer', 'question_page');
                                                                                  }
                                                                                  );

                                                                                  Post as a guest















                                                                                  Required, but never shown





















































                                                                                  Required, but never shown














                                                                                  Required, but never shown












                                                                                  Required, but never shown







                                                                                  Required, but never shown

































                                                                                  Required, but never shown














                                                                                  Required, but never shown












                                                                                  Required, but never shown







                                                                                  Required, but never shown







                                                                                  Popular posts from this blog

                                                                                  An IMO inspired problem

                                                                                  Management

                                                                                  Has there ever been an instance of an active nuclear power plant within or near a war zone?